Proving Bounded Subsets of R are Totally Bounded

In summary, to prove that a bounded subset of R is totally bounded, we can show that for any given E>0, the subset can be covered by a finite number of E-balls. We can do this by choosing the centers of the balls to be evenly spaced within the given interval and with a radius of E/2, which will cover the entire interval and more. This shows that the subset is totally bounded, meaning that for every e>0 there is a finite set that acts as an e-net for the subset.
  • #1
jdcasey9
28
0

Homework Statement



Prove that a bounded subset of R is totally bounded.


Homework Equations





The Attempt at a Solution



Fix E > 0. Let A be subset of R, x be contained in A, and B(E/2, a) where E/2 is the radius of the ball and a is the center.

Assume that B(E/2, a) is closed (since a similar open ball is contained in the closed one we can infer it is true as well). We can find balls around each x (contained in A) that can be described by B(E, x) (open or closed does not matter).

For B(E/2, a), we can always find d(x, a) <= E/2 < E...

I am having trouble describing the obvious assumption that the union of B(E, x)'s will contain (and more) all of B(E/2), do I need to use the triangle inequality? Or is a written explanation reason enough?
 
Physics news on Phys.org
  • #2
jdcasey9 said:

Homework Statement



Prove that a bounded subset of R is totally bounded.


Homework Equations





The Attempt at a Solution



Fix E > 0. Let A be subset of R, x be contained in A, and B(E/2, a) where E/2 is the radius of the ball and a is the center.

Assume that B(E/2, a) is closed (since a similar open ball is contained in the closed one we can infer it is true as well). We can find balls around each x (contained in A) that can be described by B(E, x) (open or closed does not matter).

For B(E/2, a), we can always find d(x, a) <= E/2 < E...

I am having trouble describing the obvious assumption that the union of B(E, x)'s will contain (and more) all of B(E/2), do I need to use the triangle inequality? Or is a written explanation reason enough?

That doesn't make any sense at all. Are you sure you know what the definition of 'totally bounded' is? Isn't any bounded subset of R contained in an interval like [-M,M]?
 
  • #3
Dick said:
That doesn't make any sense at all. Are you sure you know what the definition of 'totally bounded' is? Isn't any bounded subset of R contained in an interval like [-M,M]?

For A subset in R I am trying to say that A is a subset of the union of balls that contain points in A (A=B(E/2, a) every other ball centered in A is B(E, x)). I have made the radiuses as such so that the Union of balls will always encompase every point in A. So, yes, every bounded subset of R is contained in an interval like [-M, M] (or (-M, M)), my bounded interval is the closed ball B(E/2, a)

The definition from notes for totally bounded:

(M,d) Fix E>0. A subset in M is an E-net if for all x is contained in M, there exists an a contained in A such that d(x, a)<E, equivalently, for all x contained in M, d(x,a)<E, equivalently, M is a subset of the Union of B(E,a). In words: "M is covered by E-balls".
 
  • #4
jdcasey9 said:
For A subset in R I am trying to say that A is a subset of the union of balls that contain points in A (A=B(E/2, a) every other ball centered in A is B(E, x)). I have made the radiuses as such so that the Union of balls will always encompase every point in A. So, yes, every bounded subset of R is contained in an interval like [-M, M] (or (-M, M)), my bounded interval is the closed ball B(E/2, a)

The definition from notes for totally bounded:

(M,d) Fix E>0. A subset in M is an E-net if for all x is contained in M, there exists an a contained in A such that d(x, a)<E, equivalently, for all x contained in M, d(x,a)<E, equivalently, M is a subset of the Union of B(E,a). In words: "M is covered by E-balls".

That's the definition of E-net, not totally bounded. My definition says that M is totally bounded if for every e>0 there is a covering of M by a finite number of e-balls. Or, for every e>0 M contains a finite set A which is an e-net.
 
  • #5
Ok. I'm understanding the difference between the two, but I am not understanding how that changes my prospective proof.
 
Last edited:
  • #6
jdcasey9 said:
Ok. I'm understanding the difference between the two, but I am not understanding how that changes my prospective proof.

I guess I just don't understand what your proof is trying to do. On the other hand it's pretty easy to show directly that the interval [-M,M] can be covered with a finite number of balls of radius e, no matter how small e is.
 
  • #7
Ok. So,

Fix E > 0. Let A = [-M, M] which is contained in R and let x=supA, y=infA. For any E, we know that x+E, y-E are outside of our interval and x-E, y+E are inside our interval. So to find the finitely many E-balls that cover [-M, M] would we just take any points M/2, -M/2 such that balls around them cover the entire interval and more? Like defining their radius to each be half of the interval?
 
  • #8
jdcasey9 said:
Ok. So,

Fix E > 0. Let A = [-M, M] which is contained in R and let x=supA, y=infA. For any E, we know that x+E, y-E are outside of our interval and x-E, y+E are inside our interval. So to find the finitely many E-balls that cover [-M, M] would we just take any points M/2, -M/2 such that balls around them cover the entire interval and more? Like defining their radius to each be half of the interval?

You can't pick the radius of your balls, they have to be E. Suppose A is contained in [-M,M]. Then if you cover [-M,M] you cover A. To cover [-M,M] just pick the center of the balls to be -M, -M+E/2, -M+2*(E/2), -M+3*(E/2), ... How many do you need?
 
  • #9
We need enough so that the last one is centered around M. So if we continued in our pattern: -M, -M+E/2, -M+2*(E/2), -M+3*(E/2), ..., -M+2M would be last point, right? So we would need 4m/E points? (because x*(E/2) = 2M)
 
  • #10
jdcasey9 said:
We need enough so that the last one is centered around M. So if we continued in our pattern: -M, -M+E/2, -M+2*(E/2), -M+3*(E/2), ..., -M+2M would be last point, right? So we would need 4m/E points? (because x*(E/2) = 2M)

More or less, yes. E might not divide evenly into M, but the exact count isn't important. What's important is that you can clearly cover any finite interval with a finite number of balls of size E.
 
  • #11
Ok, so to prove that a bounded subset of R is totally bounded:

Let E>0. Take A, a subset of [-M, M]. We can find finitely many balls such that their union covers [-M, M] by taking the balls B[E, -M], B[E, -M + E/2], B[E, -M + E], B[E, -M + 3E/2],..., B[E, M - E/2], B[E, M] (where E is the radius and the other is the center). Therefore, we have found a finite E-net that covers the interval [-M, M] and therefore covers A. So a bounded subset of R is totally bounded.

Is that sufficient enough?
 
  • #12
jdcasey9 said:
Ok, so to prove that a bounded subset of R is totally bounded:

Let E>0. Take A, a subset of [-M, M]. We can find finitely many balls such that their union covers [-M, M] by taking the balls B[E, -M], B[E, -M + E/2], B[E, -M + E], B[E, -M + 3E/2],..., B[E, M - E/2], B[E, M] (where E is the radius and the other is the center). Therefore, we have found a finite E-net that covers the interval [-M, M] and therefore covers A. So a bounded subset of R is totally bounded.

Is that sufficient enough?

It expresses the idea well enough. If you want to make it a little neater you can also choose M to be a even multiple of E/2.
 
  • #13
Ok, but I just want to make sure that this is considered a sufficient proof for this problem. Nothing needs to be done more explicitly?
 

Related to Proving Bounded Subsets of R are Totally Bounded

1. What is a bounded subset of R?

A bounded subset of R (the set of real numbers) is a set of real numbers that is contained within a finite interval. This means that all elements in the set have a maximum and minimum value.

2. What does it mean for a bounded subset of R to be totally bounded?

A bounded subset of R is considered totally bounded if it can be covered by a finite number of open intervals (or "balls") of any given size. This essentially means that the set can be broken down into smaller and smaller subsets that are still bounded.

3. How do you prove that a bounded subset of R is totally bounded?

To prove that a bounded subset of R is totally bounded, you must show that it can be covered by a finite number of open intervals. This can be done by using the definition of a bounded set and breaking it down into smaller subsets until it can be covered by a finite number of intervals.

4. Why is it important to prove that a bounded subset of R is totally bounded?

Proving that a bounded subset of R is totally bounded is important because it is a necessary condition for other important mathematical concepts such as compactness and completeness. It also allows us to make conclusions about the behavior and properties of the set.

5. Can a subset of R be bounded but not totally bounded?

Yes, a subset of R can be bounded but not totally bounded. For example, the set of integers is bounded, but it cannot be covered by a finite number of open intervals. Therefore, it is not totally bounded.

Similar threads

  • Calculus and Beyond Homework Help
Replies
3
Views
306
Replies
4
Views
523
  • Calculus and Beyond Homework Help
Replies
1
Views
3K
  • Calculus and Beyond Homework Help
Replies
1
Views
518
  • Calculus and Beyond Homework Help
Replies
6
Views
1K
  • Calculus and Beyond Homework Help
Replies
7
Views
2K
  • Calculus and Beyond Homework Help
Replies
20
Views
2K
  • Calculus and Beyond Homework Help
Replies
1
Views
661
  • Calculus and Beyond Homework Help
Replies
3
Views
875
  • Calculus and Beyond Homework Help
Replies
7
Views
719
Back
Top